How can I prove that a linear map $T:Vto V$ such that $T^2 = I$ is equal to the Identity map?












2












$begingroup$


Suppose $T in mathcal{L}(V)$, $T^2=I$, and $-1$ is not an eigenvalue of $T$. Prove that $T=I$.



I'm not sure where to start, though I know I somehow have to show that $T circ T$ yields the identity matrix, but there isn't much information given.










share|cite|improve this question











$endgroup$












  • $begingroup$
    I eidited the title of your post to make it better conform to the problem presented in the body. Cheers!
    $endgroup$
    – Robert Lewis
    Dec 3 '18 at 23:07
















2












$begingroup$


Suppose $T in mathcal{L}(V)$, $T^2=I$, and $-1$ is not an eigenvalue of $T$. Prove that $T=I$.



I'm not sure where to start, though I know I somehow have to show that $T circ T$ yields the identity matrix, but there isn't much information given.










share|cite|improve this question











$endgroup$












  • $begingroup$
    I eidited the title of your post to make it better conform to the problem presented in the body. Cheers!
    $endgroup$
    – Robert Lewis
    Dec 3 '18 at 23:07














2












2








2


1



$begingroup$


Suppose $T in mathcal{L}(V)$, $T^2=I$, and $-1$ is not an eigenvalue of $T$. Prove that $T=I$.



I'm not sure where to start, though I know I somehow have to show that $T circ T$ yields the identity matrix, but there isn't much information given.










share|cite|improve this question











$endgroup$




Suppose $T in mathcal{L}(V)$, $T^2=I$, and $-1$ is not an eigenvalue of $T$. Prove that $T=I$.



I'm not sure where to start, though I know I somehow have to show that $T circ T$ yields the identity matrix, but there isn't much information given.







linear-algebra matrices vector-spaces eigenvalues-eigenvectors linear-transformations






share|cite|improve this question















share|cite|improve this question













share|cite|improve this question




share|cite|improve this question








edited Dec 3 '18 at 23:46









Batominovski

33.1k33293




33.1k33293










asked Dec 3 '18 at 22:34









JaigusJaigus

2259




2259












  • $begingroup$
    I eidited the title of your post to make it better conform to the problem presented in the body. Cheers!
    $endgroup$
    – Robert Lewis
    Dec 3 '18 at 23:07


















  • $begingroup$
    I eidited the title of your post to make it better conform to the problem presented in the body. Cheers!
    $endgroup$
    – Robert Lewis
    Dec 3 '18 at 23:07
















$begingroup$
I eidited the title of your post to make it better conform to the problem presented in the body. Cheers!
$endgroup$
– Robert Lewis
Dec 3 '18 at 23:07




$begingroup$
I eidited the title of your post to make it better conform to the problem presented in the body. Cheers!
$endgroup$
– Robert Lewis
Dec 3 '18 at 23:07










3 Answers
3






active

oldest

votes


















4












$begingroup$

Hint: $0 = T^2-I = (T-I)(T+I)$, with $T+I$ invertible (why?).



... or this:



Hint: $0 = T^2-I = (T+I)(T-I)$, with $T+I$ injective (why?).






share|cite|improve this answer











$endgroup$













  • $begingroup$
    I understand the expansion but, I'm not sure how to reason that $T+I$ is invertible. Since it's given that -1 is not an eigenvalue for T, is $T=I$ the only solution for $(T-I)(T+I)$ because of that?
    $endgroup$
    – Jaigus
    Dec 3 '18 at 23:04










  • $begingroup$
    @Jaigus not quite. Because $-1$ is not an eigenvalue, the only solution for $(T+I)x = 0$ is $x = 0$. That is, the matrix $T+I$ has a trivial kernel, which is to say that $T+I$ is invertible.
    $endgroup$
    – Omnomnomnom
    Dec 4 '18 at 1:05



















2












$begingroup$

Note that



$(T + I)(T - I) = T^2 - I; tag 1$



since



$T^2 = I, tag 2$



$T^2 - I = 0; tag 3$



now suppose



$T ne I; tag 4$



then



$exists ; x in v, ; Tx ne Ix = x; tag 5$



therefore,



$w = (T - I) x = Tx - x ne 0; tag 6$



thus, by (1) and (3),



$(T + I)w = (T + I)(T - I)x = (T^2 - I)x = 0(x) = 0; tag 7$



but



$(T + I)w = 0 Longrightarrow Tw + w = 0 Longrightarrow Tw = -w, tag 8$



which asserts that $w ne 0$ is an eigenvector of $T$ corresponding to eigenvalue $-1$, in contradiction to the hypothesis placed upon $T$. We see then that (5) cannot bind, whence



$forall x in v, ; Tx = Ix = x, tag 9$



as was to be proved.






share|cite|improve this answer











$endgroup$





















    1












    $begingroup$

    Assume that the base field is of characteristic not equal to $2$. First, using the hypothesis $T^2=I$ and the fact that $x-1$ and $x+1$ are coprime polynomials, show that $$V=text{im}(T+I)oplus ker(T+I),.$$ Then, prove that $text{im}(T-I)=ker(T+I)$. Since $-1$ is not an eigenvalue of $T$, this means $ker(T+I)=0$. Therefore, $text{im}(T-I)=0$, proving that $T=I$.




    Observe that, for any $vin V$, $$v=frac{1}{2}(T+I)(v)-frac{1}{2}(T-I)(v),,tag{*}$$ with $frac{1}{2}(T+I)(v)in text{im}(T+I)$ and $-frac12(T-I)(v)in text{im}(T-I)subseteq ker(T+I)$ (since $(T-I)(T+I)=T^2-I=0$). This shows that $$V=text{im}(T+I)+ker(T+I),.tag{#}$$ If $vin text{im}(T+I)cap ker(T+I)$, then $v=(T+I)(u)$ for some $uin V$, so $$(T+I)^2(u)=(T+I)(v)=0,.$$ Since $T^2-I=0$, we get $$v=(T+I)(u)=frac{1}{2}big((T+I)^2-(T^2-I)big)(u)=0,.$$ Thus, the sum (#) is direct.
    From (*), we can also see that $V=text{im}(T-I)+text{im}(T+I)$ and $text{im}(T-I)subseteq ker(T+I)$. This implies $text{im}(T-I)=ker(T+I)$.







    share|cite|improve this answer











    $endgroup$













      Your Answer





      StackExchange.ifUsing("editor", function () {
      return StackExchange.using("mathjaxEditing", function () {
      StackExchange.MarkdownEditor.creationCallbacks.add(function (editor, postfix) {
      StackExchange.mathjaxEditing.prepareWmdForMathJax(editor, postfix, [["$", "$"], ["\\(","\\)"]]);
      });
      });
      }, "mathjax-editing");

      StackExchange.ready(function() {
      var channelOptions = {
      tags: "".split(" "),
      id: "69"
      };
      initTagRenderer("".split(" "), "".split(" "), channelOptions);

      StackExchange.using("externalEditor", function() {
      // Have to fire editor after snippets, if snippets enabled
      if (StackExchange.settings.snippets.snippetsEnabled) {
      StackExchange.using("snippets", function() {
      createEditor();
      });
      }
      else {
      createEditor();
      }
      });

      function createEditor() {
      StackExchange.prepareEditor({
      heartbeatType: 'answer',
      autoActivateHeartbeat: false,
      convertImagesToLinks: true,
      noModals: true,
      showLowRepImageUploadWarning: true,
      reputationToPostImages: 10,
      bindNavPrevention: true,
      postfix: "",
      imageUploader: {
      brandingHtml: "Powered by u003ca class="icon-imgur-white" href="https://imgur.com/"u003eu003c/au003e",
      contentPolicyHtml: "User contributions licensed under u003ca href="https://creativecommons.org/licenses/by-sa/3.0/"u003ecc by-sa 3.0 with attribution requiredu003c/au003e u003ca href="https://stackoverflow.com/legal/content-policy"u003e(content policy)u003c/au003e",
      allowUrls: true
      },
      noCode: true, onDemand: true,
      discardSelector: ".discard-answer"
      ,immediatelyShowMarkdownHelp:true
      });


      }
      });














      draft saved

      draft discarded


















      StackExchange.ready(
      function () {
      StackExchange.openid.initPostLogin('.new-post-login', 'https%3a%2f%2fmath.stackexchange.com%2fquestions%2f3024796%2fhow-can-i-prove-that-a-linear-map-tv-to-v-such-that-t2-i-is-equal-to-the%23new-answer', 'question_page');
      }
      );

      Post as a guest















      Required, but never shown

























      3 Answers
      3






      active

      oldest

      votes








      3 Answers
      3






      active

      oldest

      votes









      active

      oldest

      votes






      active

      oldest

      votes









      4












      $begingroup$

      Hint: $0 = T^2-I = (T-I)(T+I)$, with $T+I$ invertible (why?).



      ... or this:



      Hint: $0 = T^2-I = (T+I)(T-I)$, with $T+I$ injective (why?).






      share|cite|improve this answer











      $endgroup$













      • $begingroup$
        I understand the expansion but, I'm not sure how to reason that $T+I$ is invertible. Since it's given that -1 is not an eigenvalue for T, is $T=I$ the only solution for $(T-I)(T+I)$ because of that?
        $endgroup$
        – Jaigus
        Dec 3 '18 at 23:04










      • $begingroup$
        @Jaigus not quite. Because $-1$ is not an eigenvalue, the only solution for $(T+I)x = 0$ is $x = 0$. That is, the matrix $T+I$ has a trivial kernel, which is to say that $T+I$ is invertible.
        $endgroup$
        – Omnomnomnom
        Dec 4 '18 at 1:05
















      4












      $begingroup$

      Hint: $0 = T^2-I = (T-I)(T+I)$, with $T+I$ invertible (why?).



      ... or this:



      Hint: $0 = T^2-I = (T+I)(T-I)$, with $T+I$ injective (why?).






      share|cite|improve this answer











      $endgroup$













      • $begingroup$
        I understand the expansion but, I'm not sure how to reason that $T+I$ is invertible. Since it's given that -1 is not an eigenvalue for T, is $T=I$ the only solution for $(T-I)(T+I)$ because of that?
        $endgroup$
        – Jaigus
        Dec 3 '18 at 23:04










      • $begingroup$
        @Jaigus not quite. Because $-1$ is not an eigenvalue, the only solution for $(T+I)x = 0$ is $x = 0$. That is, the matrix $T+I$ has a trivial kernel, which is to say that $T+I$ is invertible.
        $endgroup$
        – Omnomnomnom
        Dec 4 '18 at 1:05














      4












      4








      4





      $begingroup$

      Hint: $0 = T^2-I = (T-I)(T+I)$, with $T+I$ invertible (why?).



      ... or this:



      Hint: $0 = T^2-I = (T+I)(T-I)$, with $T+I$ injective (why?).






      share|cite|improve this answer











      $endgroup$



      Hint: $0 = T^2-I = (T-I)(T+I)$, with $T+I$ invertible (why?).



      ... or this:



      Hint: $0 = T^2-I = (T+I)(T-I)$, with $T+I$ injective (why?).







      share|cite|improve this answer














      share|cite|improve this answer



      share|cite|improve this answer








      edited Dec 4 '18 at 10:11

























      answered Dec 3 '18 at 22:36









      lhflhf

      165k10171396




      165k10171396












      • $begingroup$
        I understand the expansion but, I'm not sure how to reason that $T+I$ is invertible. Since it's given that -1 is not an eigenvalue for T, is $T=I$ the only solution for $(T-I)(T+I)$ because of that?
        $endgroup$
        – Jaigus
        Dec 3 '18 at 23:04










      • $begingroup$
        @Jaigus not quite. Because $-1$ is not an eigenvalue, the only solution for $(T+I)x = 0$ is $x = 0$. That is, the matrix $T+I$ has a trivial kernel, which is to say that $T+I$ is invertible.
        $endgroup$
        – Omnomnomnom
        Dec 4 '18 at 1:05


















      • $begingroup$
        I understand the expansion but, I'm not sure how to reason that $T+I$ is invertible. Since it's given that -1 is not an eigenvalue for T, is $T=I$ the only solution for $(T-I)(T+I)$ because of that?
        $endgroup$
        – Jaigus
        Dec 3 '18 at 23:04










      • $begingroup$
        @Jaigus not quite. Because $-1$ is not an eigenvalue, the only solution for $(T+I)x = 0$ is $x = 0$. That is, the matrix $T+I$ has a trivial kernel, which is to say that $T+I$ is invertible.
        $endgroup$
        – Omnomnomnom
        Dec 4 '18 at 1:05
















      $begingroup$
      I understand the expansion but, I'm not sure how to reason that $T+I$ is invertible. Since it's given that -1 is not an eigenvalue for T, is $T=I$ the only solution for $(T-I)(T+I)$ because of that?
      $endgroup$
      – Jaigus
      Dec 3 '18 at 23:04




      $begingroup$
      I understand the expansion but, I'm not sure how to reason that $T+I$ is invertible. Since it's given that -1 is not an eigenvalue for T, is $T=I$ the only solution for $(T-I)(T+I)$ because of that?
      $endgroup$
      – Jaigus
      Dec 3 '18 at 23:04












      $begingroup$
      @Jaigus not quite. Because $-1$ is not an eigenvalue, the only solution for $(T+I)x = 0$ is $x = 0$. That is, the matrix $T+I$ has a trivial kernel, which is to say that $T+I$ is invertible.
      $endgroup$
      – Omnomnomnom
      Dec 4 '18 at 1:05




      $begingroup$
      @Jaigus not quite. Because $-1$ is not an eigenvalue, the only solution for $(T+I)x = 0$ is $x = 0$. That is, the matrix $T+I$ has a trivial kernel, which is to say that $T+I$ is invertible.
      $endgroup$
      – Omnomnomnom
      Dec 4 '18 at 1:05











      2












      $begingroup$

      Note that



      $(T + I)(T - I) = T^2 - I; tag 1$



      since



      $T^2 = I, tag 2$



      $T^2 - I = 0; tag 3$



      now suppose



      $T ne I; tag 4$



      then



      $exists ; x in v, ; Tx ne Ix = x; tag 5$



      therefore,



      $w = (T - I) x = Tx - x ne 0; tag 6$



      thus, by (1) and (3),



      $(T + I)w = (T + I)(T - I)x = (T^2 - I)x = 0(x) = 0; tag 7$



      but



      $(T + I)w = 0 Longrightarrow Tw + w = 0 Longrightarrow Tw = -w, tag 8$



      which asserts that $w ne 0$ is an eigenvector of $T$ corresponding to eigenvalue $-1$, in contradiction to the hypothesis placed upon $T$. We see then that (5) cannot bind, whence



      $forall x in v, ; Tx = Ix = x, tag 9$



      as was to be proved.






      share|cite|improve this answer











      $endgroup$


















        2












        $begingroup$

        Note that



        $(T + I)(T - I) = T^2 - I; tag 1$



        since



        $T^2 = I, tag 2$



        $T^2 - I = 0; tag 3$



        now suppose



        $T ne I; tag 4$



        then



        $exists ; x in v, ; Tx ne Ix = x; tag 5$



        therefore,



        $w = (T - I) x = Tx - x ne 0; tag 6$



        thus, by (1) and (3),



        $(T + I)w = (T + I)(T - I)x = (T^2 - I)x = 0(x) = 0; tag 7$



        but



        $(T + I)w = 0 Longrightarrow Tw + w = 0 Longrightarrow Tw = -w, tag 8$



        which asserts that $w ne 0$ is an eigenvector of $T$ corresponding to eigenvalue $-1$, in contradiction to the hypothesis placed upon $T$. We see then that (5) cannot bind, whence



        $forall x in v, ; Tx = Ix = x, tag 9$



        as was to be proved.






        share|cite|improve this answer











        $endgroup$
















          2












          2








          2





          $begingroup$

          Note that



          $(T + I)(T - I) = T^2 - I; tag 1$



          since



          $T^2 = I, tag 2$



          $T^2 - I = 0; tag 3$



          now suppose



          $T ne I; tag 4$



          then



          $exists ; x in v, ; Tx ne Ix = x; tag 5$



          therefore,



          $w = (T - I) x = Tx - x ne 0; tag 6$



          thus, by (1) and (3),



          $(T + I)w = (T + I)(T - I)x = (T^2 - I)x = 0(x) = 0; tag 7$



          but



          $(T + I)w = 0 Longrightarrow Tw + w = 0 Longrightarrow Tw = -w, tag 8$



          which asserts that $w ne 0$ is an eigenvector of $T$ corresponding to eigenvalue $-1$, in contradiction to the hypothesis placed upon $T$. We see then that (5) cannot bind, whence



          $forall x in v, ; Tx = Ix = x, tag 9$



          as was to be proved.






          share|cite|improve this answer











          $endgroup$



          Note that



          $(T + I)(T - I) = T^2 - I; tag 1$



          since



          $T^2 = I, tag 2$



          $T^2 - I = 0; tag 3$



          now suppose



          $T ne I; tag 4$



          then



          $exists ; x in v, ; Tx ne Ix = x; tag 5$



          therefore,



          $w = (T - I) x = Tx - x ne 0; tag 6$



          thus, by (1) and (3),



          $(T + I)w = (T + I)(T - I)x = (T^2 - I)x = 0(x) = 0; tag 7$



          but



          $(T + I)w = 0 Longrightarrow Tw + w = 0 Longrightarrow Tw = -w, tag 8$



          which asserts that $w ne 0$ is an eigenvector of $T$ corresponding to eigenvalue $-1$, in contradiction to the hypothesis placed upon $T$. We see then that (5) cannot bind, whence



          $forall x in v, ; Tx = Ix = x, tag 9$



          as was to be proved.







          share|cite|improve this answer














          share|cite|improve this answer



          share|cite|improve this answer








          edited Dec 3 '18 at 23:16

























          answered Dec 3 '18 at 23:01









          Robert LewisRobert Lewis

          47k23067




          47k23067























              1












              $begingroup$

              Assume that the base field is of characteristic not equal to $2$. First, using the hypothesis $T^2=I$ and the fact that $x-1$ and $x+1$ are coprime polynomials, show that $$V=text{im}(T+I)oplus ker(T+I),.$$ Then, prove that $text{im}(T-I)=ker(T+I)$. Since $-1$ is not an eigenvalue of $T$, this means $ker(T+I)=0$. Therefore, $text{im}(T-I)=0$, proving that $T=I$.




              Observe that, for any $vin V$, $$v=frac{1}{2}(T+I)(v)-frac{1}{2}(T-I)(v),,tag{*}$$ with $frac{1}{2}(T+I)(v)in text{im}(T+I)$ and $-frac12(T-I)(v)in text{im}(T-I)subseteq ker(T+I)$ (since $(T-I)(T+I)=T^2-I=0$). This shows that $$V=text{im}(T+I)+ker(T+I),.tag{#}$$ If $vin text{im}(T+I)cap ker(T+I)$, then $v=(T+I)(u)$ for some $uin V$, so $$(T+I)^2(u)=(T+I)(v)=0,.$$ Since $T^2-I=0$, we get $$v=(T+I)(u)=frac{1}{2}big((T+I)^2-(T^2-I)big)(u)=0,.$$ Thus, the sum (#) is direct.
              From (*), we can also see that $V=text{im}(T-I)+text{im}(T+I)$ and $text{im}(T-I)subseteq ker(T+I)$. This implies $text{im}(T-I)=ker(T+I)$.







              share|cite|improve this answer











              $endgroup$


















                1












                $begingroup$

                Assume that the base field is of characteristic not equal to $2$. First, using the hypothesis $T^2=I$ and the fact that $x-1$ and $x+1$ are coprime polynomials, show that $$V=text{im}(T+I)oplus ker(T+I),.$$ Then, prove that $text{im}(T-I)=ker(T+I)$. Since $-1$ is not an eigenvalue of $T$, this means $ker(T+I)=0$. Therefore, $text{im}(T-I)=0$, proving that $T=I$.




                Observe that, for any $vin V$, $$v=frac{1}{2}(T+I)(v)-frac{1}{2}(T-I)(v),,tag{*}$$ with $frac{1}{2}(T+I)(v)in text{im}(T+I)$ and $-frac12(T-I)(v)in text{im}(T-I)subseteq ker(T+I)$ (since $(T-I)(T+I)=T^2-I=0$). This shows that $$V=text{im}(T+I)+ker(T+I),.tag{#}$$ If $vin text{im}(T+I)cap ker(T+I)$, then $v=(T+I)(u)$ for some $uin V$, so $$(T+I)^2(u)=(T+I)(v)=0,.$$ Since $T^2-I=0$, we get $$v=(T+I)(u)=frac{1}{2}big((T+I)^2-(T^2-I)big)(u)=0,.$$ Thus, the sum (#) is direct.
                From (*), we can also see that $V=text{im}(T-I)+text{im}(T+I)$ and $text{im}(T-I)subseteq ker(T+I)$. This implies $text{im}(T-I)=ker(T+I)$.







                share|cite|improve this answer











                $endgroup$
















                  1












                  1








                  1





                  $begingroup$

                  Assume that the base field is of characteristic not equal to $2$. First, using the hypothesis $T^2=I$ and the fact that $x-1$ and $x+1$ are coprime polynomials, show that $$V=text{im}(T+I)oplus ker(T+I),.$$ Then, prove that $text{im}(T-I)=ker(T+I)$. Since $-1$ is not an eigenvalue of $T$, this means $ker(T+I)=0$. Therefore, $text{im}(T-I)=0$, proving that $T=I$.




                  Observe that, for any $vin V$, $$v=frac{1}{2}(T+I)(v)-frac{1}{2}(T-I)(v),,tag{*}$$ with $frac{1}{2}(T+I)(v)in text{im}(T+I)$ and $-frac12(T-I)(v)in text{im}(T-I)subseteq ker(T+I)$ (since $(T-I)(T+I)=T^2-I=0$). This shows that $$V=text{im}(T+I)+ker(T+I),.tag{#}$$ If $vin text{im}(T+I)cap ker(T+I)$, then $v=(T+I)(u)$ for some $uin V$, so $$(T+I)^2(u)=(T+I)(v)=0,.$$ Since $T^2-I=0$, we get $$v=(T+I)(u)=frac{1}{2}big((T+I)^2-(T^2-I)big)(u)=0,.$$ Thus, the sum (#) is direct.
                  From (*), we can also see that $V=text{im}(T-I)+text{im}(T+I)$ and $text{im}(T-I)subseteq ker(T+I)$. This implies $text{im}(T-I)=ker(T+I)$.







                  share|cite|improve this answer











                  $endgroup$



                  Assume that the base field is of characteristic not equal to $2$. First, using the hypothesis $T^2=I$ and the fact that $x-1$ and $x+1$ are coprime polynomials, show that $$V=text{im}(T+I)oplus ker(T+I),.$$ Then, prove that $text{im}(T-I)=ker(T+I)$. Since $-1$ is not an eigenvalue of $T$, this means $ker(T+I)=0$. Therefore, $text{im}(T-I)=0$, proving that $T=I$.




                  Observe that, for any $vin V$, $$v=frac{1}{2}(T+I)(v)-frac{1}{2}(T-I)(v),,tag{*}$$ with $frac{1}{2}(T+I)(v)in text{im}(T+I)$ and $-frac12(T-I)(v)in text{im}(T-I)subseteq ker(T+I)$ (since $(T-I)(T+I)=T^2-I=0$). This shows that $$V=text{im}(T+I)+ker(T+I),.tag{#}$$ If $vin text{im}(T+I)cap ker(T+I)$, then $v=(T+I)(u)$ for some $uin V$, so $$(T+I)^2(u)=(T+I)(v)=0,.$$ Since $T^2-I=0$, we get $$v=(T+I)(u)=frac{1}{2}big((T+I)^2-(T^2-I)big)(u)=0,.$$ Thus, the sum (#) is direct.
                  From (*), we can also see that $V=text{im}(T-I)+text{im}(T+I)$ and $text{im}(T-I)subseteq ker(T+I)$. This implies $text{im}(T-I)=ker(T+I)$.








                  share|cite|improve this answer














                  share|cite|improve this answer



                  share|cite|improve this answer








                  edited Dec 4 '18 at 0:03

























                  answered Dec 3 '18 at 23:37









                  BatominovskiBatominovski

                  33.1k33293




                  33.1k33293






























                      draft saved

                      draft discarded




















































                      Thanks for contributing an answer to Mathematics Stack Exchange!


                      • Please be sure to answer the question. Provide details and share your research!

                      But avoid



                      • Asking for help, clarification, or responding to other answers.

                      • Making statements based on opinion; back them up with references or personal experience.


                      Use MathJax to format equations. MathJax reference.


                      To learn more, see our tips on writing great answers.




                      draft saved


                      draft discarded














                      StackExchange.ready(
                      function () {
                      StackExchange.openid.initPostLogin('.new-post-login', 'https%3a%2f%2fmath.stackexchange.com%2fquestions%2f3024796%2fhow-can-i-prove-that-a-linear-map-tv-to-v-such-that-t2-i-is-equal-to-the%23new-answer', 'question_page');
                      }
                      );

                      Post as a guest















                      Required, but never shown





















































                      Required, but never shown














                      Required, but never shown












                      Required, but never shown







                      Required, but never shown

































                      Required, but never shown














                      Required, but never shown












                      Required, but never shown







                      Required, but never shown







                      Popular posts from this blog

                      Biblatex bibliography style without URLs when DOI exists (in Overleaf with Zotero bibliography)

                      ComboBox Display Member on multiple fields

                      Is it possible to collect Nectar points via Trainline?